10
$\begingroup$

Let $G$ be a $3$-connected Hamiltonian graph with at least one edge that belongs to each H-cycle of $G$. Some authors (e.g. in the link given here) call such an edge an a-edge and an edge that belongs to no H-cycle of $G$ a b-edge. Let $a(G)$ and $b(G)$ denote the number of a-edges and b-edges, respectively. Define $\rho(G)=\dfrac{b(G)}{a(G)} $.

  • Are there good (or even sharp) upper bounds for $\rho(G)$ ? Maybe $\rho(G)<1$?

Such graphs can be constructed e.g. using certain non-Hamiltonian graphs like the so-called Tutte fragment $T$ or the “Petersen fragment” $P$, the latter obtained from the Petersen graph by adding a vertex on each of three consecutive edges and an edge leaving at each of those vertices. These edges are labelled by $1,2,3$.

T & P
It is easy to see that $T$ does not admit a H-path between 1 and 3, whereas $P$ admits no H-path starting at 2, only two H-paths between 1 and 3 (one consists of the violet and the red edges, the other one of the violet and the black edges). $T$ has two a-edges (marked in violet) and no b-edges; $P$ has nine a-edges (violet) and three b-edges (light blue).
The simplest $3$-connected Hamiltonian graphs obtained from these are a prism over the big “triangle” of $T$ with $a(G)=3$ and $b(G)=0$ (note that this one is even planar) and likewise a prism over $P$, which has $a(G)=16$ and $b(G)=5$.

By taking a Hamiltonian graph and replacing certain cubic vertices by $T$ or $P$, we can impose restrictions on the H-cycles and obtain graphs with various values of $a(G)$ and $b(G)$.
If we start with the wheel $W_{n}$ and replace one vertex by a $T$ in such a way that the spike becomes an a-edge, the resulting graph 'keeps' only two of the H-cycles of $W_n$ and has $\rho(G)=\dfrac{n-3}{n}$. I wonder if this is best possible. So if generally $\rho(G)<1$ holds, that would be sharp.

What about the maximum if we consider only cubic graphs ?
If we start with the prism over $K_3$ and make one ‘vertical' edge a b-edge by replacing one vertex with a $P$, we get a graph with a unique$^*$ H-cycle and $\rho(G)=\dfrac{5}{13}\approx.3846 $.
If we start with the dodecahedron and 'block' three well chosen edges (i.e. make them into b-edges) by replacing three vertices with $P$'s, we get a graph with a unique$^*$ H-cycle and $\rho(G)=\dfrac{16}{41}\approx.3902 $, slightly larger.
Starting with a truncated icosahedron (soccerball), we can still do better. It depends on how many edges have to be blocked by using $P$'s to remain with a unique$^*$ H-cycle: if there are $k$ such edges and the resulting graph $G$ is unique$^*$ Hamiltonian, it will have $\rho(G)=\dfrac{30+2k}{60+7k} $, which is equal to $\dfrac{16}{41}$ for $k=9$ and gets bigger as $k$ decreases. I've checked that $k=7$, thus $\rho(G)=\dfrac{44}{109}\approx.4037 $, is possible. (Note that in the drawing given in the link, only the green and black 1-factors yield a H-cycle.)

$^*$ Edit: "unique" is meant w.r.t. the H-cycles of the original graph, because inside the instances of $P$ there are of course several ways of making H-paths through them.

Of course I don't claim that this method yields the best possible results. So the question:

  • Are there cubic, 3-connected graphs with $\rho(G)$ even bigger than that?
$\endgroup$

2 Answers 2

4
$\begingroup$

The first counterexample given by @joro gives rise to the following family $G_k$ of graphs with $\rho(G_k)=k$ : $G_k$ has $n=4k+3$ vertices numbered $0,1,\dots,n-1$ forming a H-cycle (imagine a regular n-gon embedded such that the vertex $2k+1$ is on the bottom), plus the ‘horizontal’ chords $(2j-1,4k+3-2j)$ for $j=1,...,k$ (call their set $B$) and the other chords $(j,2k+1+j)$ for $j=0,...,2k+1$. It is not hard to see that $(4k+2,0)$ is an a-edge. As the graph without this edge and without $B$ is bipartite, all edges of $B$ must be b-edges. It remains to check that for each other edge, there is a H-cycle that doesn’t contain it.

For the second question, as @Martin Tancer and @nvcleemp have pointed aout, $\rho(G)\le1/2$ for cubic graphs. The following construction comes arbitrarily close to $1/2$ :

For $n=2k$, start with a H-cycle $0,...,2k-1,0$ and add the ‘horizontal’ chords $(j,2k-j)$ for $j=1,...,k-1$ and the ‘vertical’ chord $(0,k)$. Replace the vertex $k$ with an instance of $P$ that blocks the vertical chord. Then it is easy to see (starting at the vertex $0$) that the horizontal chords cannot be part of a H-cycle. So this graph has $a(G)=2k+7$ (the original cycle plus 7 inner edges of $P$) and $b(G)=k+1$ (the horizontal chords plus 2 inner edges of $P$).

So both original questions are answered, thanks to your input. Remains the interesting question raised by joro what can be said about $\rho(G)$ for 4-regular graphs.

$\endgroup$
3
  • $\begingroup$ You mean the chords are modulo n? Verified with program for small $k$. $\endgroup$
    – joro
    Dec 27, 2013 at 8:59
  • $\begingroup$ Yes modulo n, but the way I've written it, you don't even need that. I don't have time right now to add a diagram for say k=2 or k=3, but you can easily draw one. For checking that the edges (i,i+1) for i=0,...,n-2 aren't a-edges, it suffices to note that there are 2 H-cycles (symmetric to each other w.r.t. the vertical axis) which alternate essentially between diagonals of the n-gon and edges of it. $\endgroup$
    – Wolfgang
    Dec 27, 2013 at 9:55
  • $\begingroup$ OK. Here is a drawing for k=3: s27.postimg.org/wwx38t3w3/graph_k_3.png $\endgroup$
    – joro
    Dec 27, 2013 at 10:04
4
$\begingroup$

The computer found counterexamples to $\rho(G)<1$.

Despite verification, I am not sure this is correct.

$G_1$ on $7$ vertices, $G_2$ on $11$ vertices. $\rho(G_1)=1,\rho(G_2)=2$

$G_1$:

edges=[(0, 3), (0, 4), (0, 5), (1, 4), (1, 5), (1, 6), (2, 4), (2, 5), (2, 6), (3, 5), (3, 6), (4, 6)]
A=[(0, 3)]
B=[(4, 6)]

$G_2$:

edges=[(0, 5), (0, 6), (0, 7), (1, 6), (1, 7), (1, 8), (2, 6), (2, 8), (2, 9), (3, 7), (3, 9), (3, 10), (4, 8), (4, 9), (4, 10), (5, 9), (5, 10), (6, 8), (7, 10)]
A=[(0, 5)]
B=[(6, 8), (7, 10)]

Plot of $G_1$:

G_1

$\endgroup$
24
  • $\begingroup$ Thank you. Why should this not be correct? There are interesting patterns in your 2 graphs, which might allow (at least worth a try) to restrict further searches for even higher $\rho(G)$'s: Put $V=V_3\cup V_4$ (vertices of degree 3 and 4). Did you notice that G1 and G2 are "almost bipartite", the vertices of $V_3$ essentially alternating in each H-cycle with those of $V_4$. At the exception of 03 (05 for G2), which is... an a-edge! And the only edges in $\langle V_4\rangle$ are the b-edges. H-cycle e.g. for G1: 0 4 2 6 1 5 3, for G2: 0 6 2 8 1 7 3 9 4 10 5. $\endgroup$
    – Wolfgang
    Dec 21, 2013 at 13:25
  • $\begingroup$ @Wolfgang if you trust my program, there are no cubic counterexamples rho(G) > 1 on less than 20 vertices. $\endgroup$
    – joro
    Dec 21, 2013 at 13:38
  • $\begingroup$ Also note that displaying your $G_1$ and $G_2$ like you would display bipartite graphs, it is almost immediately clear why the a-edges and b-edges are what they are. BTW do you have a feeling that $\rho(G)$ is bounded? $\endgroup$
    – Wolfgang
    Dec 21, 2013 at 13:39
  • $\begingroup$ For cubic ones, I conjecture $rho(G)\le 1/2$. Have you found cubic graphs with bigger rho? $\endgroup$
    – Wolfgang
    Dec 21, 2013 at 13:41
  • 2
    $\begingroup$ Isn't $\rho(G) \leq 1/2$ somewhat trivial? A $b$-edge in a cubic graph is incident to four $a$-edges. Each $a$-edge comes this way from at most two $b$-edges. $\endgroup$ Dec 21, 2013 at 15:08

Your Answer

By clicking “Post Your Answer”, you agree to our terms of service and acknowledge that you have read and understand our privacy policy and code of conduct.

Not the answer you're looking for? Browse other questions tagged or ask your own question.